Доказательство того, что (1/2,1/2)(1/2,1/2)(1/2,1/2) представление группы Лоренца является 4-векторным

Взято из Квантовой теории поля Зи, задача II.3.1:

Покажите явным вычислением, что ( 1 2 , 1 2 ) действительно является вектором Лоренца.

Это было задано здесь:

Как мне построить С U ( 2 ) представление Группы Лоренца с использованием С U ( 2 ) × С U ( 2 ) С О ( 3 , 1 ) ?

но я не могу переварить формальность этого ответа, имея лишь небольшое знание групп и представлений.

Играя с генераторами группы Лоренца, можно найти основу Дж ± я которые по отдельности имеют алгебру Ли С U ( 2 ) , и, таким образом, могут быть отдельно заданы спиновые представления.

Мой подход состоял в том, чтобы написать

Дж + я "=" 1 2 ( Дж я + я К я ) "=" 1 2 о я
Дж я "=" 1 2 ( Дж я я К я ) "=" 1 2 о я
что подразумевает, что
Дж я "=" о я
К я "=" 0
Однако я действительно не понимаю, куда идти дальше.

Связанный с этим вопрос от OP: physics.stackexchange.com/q/321276/2451 и ссылки в нем.
Изоморфизм можно увидеть, поняв, что матрицы о α β ˙ мю составляет основу всех 2 × 2 матрицы, поэтому произвольная матрица А α β ˙ можно записать как А α β ˙ "=" А мю о α β ˙ мю . LHS этого трансформируется в ( 0 , 1 2 ) ( 1 2 , 0 ) "=" ( 1 2 , 1 2 ) . Это уравнение говорит вам, каково изменение базиса между обычным векторным базисом А мю и ( 1 2 , 1 2 ) основа.

Ответы (3)

Сначала вспомним, как строить конечномерные неприводимые представления группы Лоренца. Сказать Дж я три генератора вращения и К я это три наддувных генератора.

л Икс "=" ( 0 0 0 0 0 0 0 0 0 0 0 1 0 0 1 0 ) л у "=" ( 0 0 0 0 0 0 0 1 0 0 0 0 0 1 0 0 ) л г "=" ( 0 0 0 0 0 0 1 0 0 1 0 0 0 0 0 0 ) К Икс "=" ( 0 1 0 0 1 0 0 0 0 0 0 0 0 0 0 0 ) К у "=" ( 0 0 1 0 0 0 0 0 1 0 0 0 0 0 0 0 ) К г "=" ( 0 0 0 1 0 0 0 0 0 0 0 0 1 0 0 0 )
Они удовлетворяют
[ Дж я , Дж Дж ] "=" ε я Дж к Дж к [ К я , К Дж ] "=" ε я Дж к Дж к [ Дж я , К Дж ] "=" ε я Дж к К к .
(Обратите внимание, что я использую кососопряженное соглашение для элементов алгебры Ли, где я не умножал на я .)

Затем мы определяем

А я "=" 1 2 ( Дж я я К я ) Б я "=" 1 2 ( Дж я + я К я )
которые удовлетворяют коммутационным соотношениям
[ А я , А Дж ] "=" ε я Дж к А к [ Б я , Б Дж ] "=" ε я Дж к Б к [ А я , Б Дж ] "=" 0.

Вот как вы строите представление группы Лоренца: сначала выберите два неотрицательных полуцелых числа Дж 1 и Дж 2 . Они соответствуют двум спинам Дж представления с ты ( 2 ) , который я обозначу

π Дж .
Напомним, что
с ты ( 2 ) "=" с п а н р { я 2 о Икс , я 2 о у , я 2 о г }
где
[ я 2 о я , я 2 о Дж ] "=" я 2 ε я Дж к о к .
Для этого вопроса нам нужно знать только спин 1 / 2 представительство с ты ( 2 ) , который дается
π 1 2 ( я 2 о я ) "=" я 2 о я .

Хорошо, как мы построим ( Дж 1 , Дж 2 ) представление группы Лоренца? Любой элемент алгебры Ли Икс е с о ( 1 , 3 ) можно записать в виде линейной комбинации А я и Б я :

Икс "=" я "=" 1 3 ( α я А я + β я Б я ) .
(Обратите внимание, что на самом деле мы имеем дело с комплексной версией алгебры Ли с о ( 1 , 3 ) потому что наши определения А я и Б я иметь факторы я , так α , β е С .)

А я и Б я сформировать свои независимые с ты ( 2 ) алгебры.

Представление алгебры Ли π ( Дж 1 , Дж 2 ) затем дается

π ( Дж 1 , Дж 2 ) ( Икс ) "=" π ( Дж 1 , Дж 2 ) ( α я А я + β Дж Б я ) π Дж 1 ( α я А я ) ( π Дж 2 ( β Дж Б я ) ) *
где звездочка обозначает комплексное сопряжение.

Иногда забывают упомянуть, что надо включать комплексное сопряжение, а иначе не получится!

Если Дж 1 "=" 1 / 2 и Дж 2 "=" 1 / 2 , у нас есть

π 1 2 ( А я ) "=" я 2 о я я ( π 1 2 ( Б я ) ) * "=" я 2 я о я * .
Мы можем явно записать эти тензорные произведения в терминах 2 × 2 "=" 4 размерная основа. (Здесь я использую для этого так называемое « произведение Кронекера ». Это просто причудливое название для умножения всех элементов двух 2 × 2 по ячейкам, чтобы получить 4 × 4 матрица.)
π ( 1 2 , 1 2 ) ( А Икс ) "=" я 2 ( 0 0 1 0 0 0 0 1 1 0 0 0 0 1 0 0 ) ( π ( 1 2 , 1 2 ) ( Б Икс ) ) * "=" я 2 ( 0 1 0 0 1 0 0 0 0 0 0 1 0 0 1 0 ) π ( 1 2 , 1 2 ) ( А у ) "=" 1 2 ( 0 0 1 0 0 0 0 1 1 0 0 0 0 1 0 0 ) ( π ( 1 2 , 1 2 ) ( Б у ) ) * "=" 1 2 ( 0 1 0 0 1 0 0 0 0 0 0 1 0 0 1 0 ) π ( 1 2 , 1 2 ) ( А г ) "=" я 2 ( 1 0 0 0 0 1 0 0 0 0 1 0 0 0 0 1 ) ( π ( 1 2 , 1 2 ) ( Б г ) ) * "=" я 2 ( 1 0 0 0 0 1 0 0 0 0 1 0 0 0 0 1 )
Затем мы можем выписать матрицы вращений и бустов Дж я и К я с использованием
Дж я "=" А я + Б я К я "=" я ( А я Б я ) .
π ( 1 2 , 1 2 ) ( Дж Икс ) "=" я 2 ( 0 1 1 0 1 0 0 1 1 0 0 1 0 1 1 0 ) π ( 1 2 , 1 2 ) ( К Икс ) "=" 1 2 ( 0 1 1 0 1 0 0 1 1 0 0 1 0 1 1 0 ) π ( 1 2 , 1 2 ) ( Дж у ) "=" 1 2 ( 0 1 1 0 1 0 0 1 1 0 0 1 0 1 1 0 ) π ( 1 2 , 1 2 ) ( К у ) "=" я 2 ( 0 1 1 0 1 0 0 1 1 0 0 1 0 1 1 0 ) π ( 1 2 , 1 2 ) ( Дж г ) "=" ( 0 0 0 0 0 я 0 0 0 0 я 0 0 0 0 0 ) π ( 1 2 , 1 2 ) ( К г ) "=" ( 1 0 0 0 0 0 0 0 0 0 0 0 0 0 0 1 )
Это странные матрицы, хотя мы можем заставить их выглядеть гораздо более наводящими на размышления в другом базисе. Определите матрицу
U "=" 1 2 ( 1 0 0 1 0 1 я 0 0 1 я 0 1 0 0 1 ) .
Удивительно,
U 1 ( π ( 1 2 , 1 2 ) ( л я ) ) U "=" л я U 1 ( π ( 1 2 , 1 2 ) ( К я ) ) U "=" К я .
Следовательно ( 1 2 , 1 2 ) представление эквивалентно обычному «векторному» представлению С О + ( 1 , 3 ) . Однако эти «векторы» живут в С 4 , нет р 4 , о чем обычно не упоминают.

@ user1379857 Пожалуйста, ознакомьтесь с этим предложенным изменением - я отклонил его, поскольку маловероятно, что кто-то, не относящийся к сообщению, может полностью оценить, применимы ли указанные ошибки, но они вполне могут нуждаться в исправлении.
(Кто бы ни предложил анонимное редактирование: это выходит за рамки уровня незначительного редактирования, подходящего здесь. Для этого типа исправления нужно прокомментировать сообщение и дать автору возможность исправить любые проблемы или уточнить, почему это прямо как есть)
Ах да, редактирование было на самом деле правильным, у меня была неправильная форма U. Я принял это, и теперь это должно быть правильно.
Почему для тензорного произведения представлений требуется комплексное сопряжение? Кроме того, где была получена матрица U? Это не нормальная матрица, к которой я привык из-за добавления угловых моментов. Спасибо.
@EmilioPisanty Не могли бы вы объяснить, почему у Ai и Bi должны быть одинаковые углы в ответе? Разве это не имеет значения? Разве они не могут изменяться независимо?

Извините, что этот ответ пришел так долго после вашего поста. Поскольку это задача типа домашнего задания, я не буду выполнять все расчеты, но распишу все ключевые моменты; в любом случае, вероятно, проще следовать этому пути, если вы сами заполняете матричную алгебру!

Чтобы понять, что здесь происходит, прежде всего подумайте о системе со спином 1. В таком случае действие Дж 3 на векторах представляется матрицей

( 0 я 0 я 0 0 0 0 0 )
Если вы будете следовать стандартной конструкции представления со спином 1 с использованием лестничных операторов, которые можно найти в любом учебнике по КМ, вместо этого вы обнаружите, что Дж 3 представлен матрицей
( 1 0 0 0 0 0 0 0 1 )
потому что он действует на основе Дж 3 собственные состояния. Вы можете видеть, что эти матрицы унитарно эквивалентны, найдя собственные значения первой из них равными 1 , 0 , 1 ; разработка соответствующих собственных векторов дает вам матрицу, которая меняет базис (с точностью до некоторого порядка этого базиса). (Приятно заметить, что эта матрица преобразует линейно поляризованные поперечные векторные поля в циркулярно поляризованные!)

Теперь в данном случае, когда мы имеем ( 1 / 2 , 1 / 2 ) "=" С U ( 2 ) С U ( 2 ) . Дж 3 действует как на (1/2,0), так и на (0,1/2) представления 1 2 о 3 . Поскольку это генератор группы, а не элемент группы, действие на тензорное произведение двух групп определяется выражением

Дж 3 "=" 1 2 о 3 я + я 1 2 о 3 "=" ( 1 0 0 0 0 0 0 0 0 0 0 0 0 0 0 1 )
Ясно, что эта матрица дает нам действие Дж 3 на его собственных состояниях, а замена базиса дает нам 4-векторное представление. Разница в том, что теперь у нас есть два собственных значения 0; они возникли в результате «сложения углового момента» наших двух представлений со спином 1/2. Один из повторений со вращением 1, как и раньше. Другой - из представления со спином 0; это ваше направление времени в 4-векторе, который является скаляром при вращении. Таким образом, в этом случае компоненты матрицы, которая изменяет действие базиса на направления 0 собственных значений, являются коэффициентами Клебша-Гордона.

Надеюсь, это поможет.

Вы неправильно поняли стратегию. Идея состоит в том, что мы определяем новые операторы А я "=" Дж я + я К я 2 и Б я "=" Дж я я К я 2 . Теперь вычислите [ А я , А Дж ] , [ Б я , Б Дж ] и [ А я , Б Дж ] Ответ, который вы должны получить, состоит в том, что первые два должны дать вам алгебру лжи С U ( 2 ) в то время как второй должен дать вам ноль. Это означает, что у нас есть, по крайней мере, локально С U ( 2 ) × С U ( 2 )

Тогда лоренцевы бусты для двухкомпонентного спинора равны е я о θ + о ф а другой е я о θ о ф . Прямая сумма этого делает общую работу, т.е. ( е я о θ + о ф 0 0 е я о θ о ф )

Таким образом, вы сможете получить генераторы, которые хотите.

Это то, что было доказано в книге. Я думаю, мне нужно явно вычислить образующие группы Лоренца, взяв тензорное произведение двух С U ( 2 ) представления. Как мне это сделать?
Я добавил, как выглядят усилители Лоренца, чтобы было ясно, что такое генераторы.
Я полагаю, что вы действительно неправильно поняли вопрос. Ваши ответы относятся к представлению спинора Дирака, которое представляет собой прямую сумму (1/2, 0) + (0,1/2). ОП запрашивает эквивалентность представления (1/2, 1/2) фундаментальному 4-векторному невозврату.